0 Daumen
351 Aufrufe

Aufgabe:

$$ \sum \limits_{n=1}^{\infty} \frac{1}{4^n}\cdot\frac{n+2}{n^2+2} \cdot(x-4)^n $$


Problem/Ansatz:

Kann mir jemand dazu die Konvergenz bestimmen?

Ich habe es umgeschrieben und mit dem Wurzelkriterium versucht, aber ich komm im Nenner dann auf 0.

Bitte ruhig ausführlich, wenns geht.


Vielen Dank im Voraus.

Avatar von

1 Antwort

+1 Daumen
 
Beste Antwort

Aloha :)

Zur Bestimmung des Konvergenzradius \(r\) sind nur die Koeffizienten \(a_n=\frac{1}{4^n}\cdot\frac{n+2}{n^2+2}\) von Interesse:

$$r=\lim\limits_{n\to\infty}\left|\frac{a_n}{a_{n+1}}\right|=\lim\limits_{n\to\infty}\left|\frac{\frac{1}{4^n}\cdot\frac{n+2}{n^2+2}}{\frac{1}{4^{n+1}}\cdot\frac{(n+1)+2}{(n+1)^2+2}}\right|=4\lim\limits_{n\to\infty}\left|\frac{n+2}{n^2+2}\cdot\frac{(n+1)^2+2}{(n+1)+2}\right|$$$$\phantom{r}=4\lim\limits_{n\to\infty}\left|\frac{(n+2)(n^2+2n+3)}{(n^2+2)(n+3)}\right|=4\lim\limits_{n\to\infty}\left|\frac{\frac{(n+2)}{n}\frac{(n^2+2n+3)}{n^2}}{\frac{(n^2+2)}{n^2}\frac{(n+3)}{n}}\right|$$$$\phantom{r}=4\lim\limits_{n\to\infty}\left|\frac{(1+\frac{2}{n})(1+\frac{2}{n}+\frac{3}{n^2})}{(1+\frac{2}{n^2})(1+\frac{3}{n})}\right|=4$$

Avatar von 148 k 🚀

Ich bearbeite zufällig gerade die Aufgabe nochmal, da ich in 2Wochen die Prüfung dazu habe. (Erste leider nicht bestanden)

In der Aufgabenstellung steht weiter: "Zeige, für welche x es konvergiert. "

Meine Antwort:

x ist im offenem Intervall von (4-1,4+1)=(3,5) konvergent.


Wäre dies so richtig?

Zur Bestimmung des Konvergenzradius \(r\) betrachtet man nur die Koeffizienten \(a_n\). Bei der Interpretation des Konvergenzradius \(r\) kommt dann die "Verschiebung" \(x_0\) ins Spiel. Die Potenzreihe konvergiert für alle \(x\), für die gilt:$$|x-x_0|<r$$Im vorliegenden Fall ist \(x_0=4\) und \(r=4\), d.h.$$|x-4|<4$$$$-4<x-4<4$$$$0<x<8$$Die Potenzreihe konvergiert also sicher für \(x\in]0;8[\).

Je nach Aufgabenstellung kann es sein, dass du die "Ränder" \(0\) bzw. \(8\) noch explizit auf Konvergenz untersuchen musst. Das scheint hier aber nicht verlangt zu sein.

Ok, Vielen dank! Habs verstanden :-)

Ein anderes Problem?

Stell deine Frage

Willkommen bei der Mathelounge! Stell deine Frage einfach und kostenlos

x
Made by a lovely community